Difference between revisions of "1957 AHSME Problems/Problem 21"

Line 1: Line 1:
 +
== Problem 21==
 +
 +
Start with the theorem "If two angles of a triangle are equal, the triangle is isosceles," and the following four statements:
 +
 +
1. If two angles of a triangle are not equal, the triangle is not isosceles.
 +
2. The base angles of an isosceles triangle are equal.
 +
3. If a triangle is not isosceles, then two of its angles are not equal.
 +
4. A necessary condition that two angles of a triangle be equal is that the triangle be isosceles.
 +
 +
Which combination of statements contains only those which are logically equivalent to the given theorem?
 +
 +
<math>\textbf{(A)}\ 1,\,2,\,3,\,4 \qquad \textbf{(B)}\ 1,\,2,\,3\qquad \textbf{(C)}\ 2,\,3,\,4\qquad \textbf{(D)}\ 1,\,2\qquad\textbf{(E)}\ 3,\,4  </math> 
 +
 
(1) is the inverse
 
(1) is the inverse
 
(2) is the converse
 
(2) is the converse

Revision as of 15:30, 10 June 2024

Problem 21

Start with the theorem "If two angles of a triangle are equal, the triangle is isosceles," and the following four statements:

1. If two angles of a triangle are not equal, the triangle is not isosceles. 2. The base angles of an isosceles triangle are equal. 3. If a triangle is not isosceles, then two of its angles are not equal. 4. A necessary condition that two angles of a triangle be equal is that the triangle be isosceles.

Which combination of statements contains only those which are logically equivalent to the given theorem?

$\textbf{(A)}\ 1,\,2,\,3,\,4 \qquad \textbf{(B)}\ 1,\,2,\,3\qquad \textbf{(C)}\ 2,\,3,\,4\qquad \textbf{(D)}\ 1,\,2\qquad\textbf{(E)}\ 3,\,4$

(1) is the inverse (2) is the converse (3) is the contrapositive (4) is a restatement of the original conditional Therefore, (3) and (4) are correct. $\boxed{\textbf{(E) } (3), (4)}$